Vous êtes sur la page 1sur 72

Exam Title : Test Series 3

Email :

Exam URL : http://test.iasbaba.com

1. Consider the following statements with regard to Government of India Act 1909
1. It introduced diarchy in provinces.
2. For the first time it provided for the association of Indians with the executive councils of viceroy.
3. It introduced the system of separate electorate by giving communal representation to Muslims.
Choose the correct answer from the codes given below

a) 1 and 2
b) 2 and 3
c) 1 and 3
d) 1, 2 and 3
Correct Answer: B
Your Answer: Unanswered
Explanation:
Solution (b)

· Important provisions of GOI act 1909 or Morley Minto reforms

· It increased the size of legislative councils from 16 to 60.

· It provided for association of Indians with executive council of viceroy. Satyendra Prasad sinha became
the first Indian to join the viceroy’s executive council.

· It introduced system of communal representation for Muslims by accepting concept of separate


electorate.

· It enlarged the deliberative functions of legislative councils like asking supplementary questions,
moving resolutions on budget etc.

· Diarchy in provinces is a provision of GOI ACT 1919

2. Consider the following


1. Hepatitis A
2. SARS
3. Typhoid
4. Cholera
Which of the above is/are water borne disease?

a) 1, 3 and 4
b) 3 and 4
c) 2, 3 and 4
d) All
Correct Answer: D

IASbaba
Web: http://www.iasbaba.com/ Page 1
Email: iasbabailp2016@gmail.com
Exam Title : Test Series 3

Email :

Exam URL : http://test.iasbaba.com

Your Answer: Unanswered


Explanation:
Solution (d)

All of the above are Water borne diseases.

https://en.wikipedia.org/wiki/Waterborne_diseases#Bacterial_infections

Hint- Science Class 7 th - Chapter 18

3. Which of the following is not a type of silk?

a) Mulberry silk
b) Tassar silk
c) Mooga silk
d) Moth silk
Correct Answer: D
Your Answer: Unanswered
Explanation:
Solution (d)

NCERT Science 7 th - Chapter 3

4. In Preamble, the ideal of justice–social, economic and political has been take from

a) American revolution
b) Revolution for unification of Germany
c) Russian revolution
d) Meiji restoration
Correct Answer: C
Your Answer: Unanswered
Explanation:
Solution (c)

· The ideal of justice—social, economic and political—has been taken from the Russian Revolution
(1917).

· The term ‘justice’ in the Preamble embraces three distinct forms—social, economic and political,
secured through various provisions of Fundamental Rights and Directive Principles.

· Source: Laxmikanth Chapter 4

IASbaba
Web: http://www.iasbaba.com/ Page 2
Email: iasbabailp2016@gmail.com
Exam Title : Test Series 3

Email :

Exam URL : http://test.iasbaba.com

5. Consider the following statements regarding India’s Foreign Trade Policy 2015-20
1. It aims at increasing India’s exports in goods and services to 900 billion by 2019-20.
2. Previous reward schemes of goods and services for export have been merged into two schemes-
Merchandise Export from India Scheme (MEIS) and Service Export from India Scheme (SEIS).
3. It seeks to raise India’s share in World exports from 2% to 6%.
Select the correct option

a) 1 only
b) 2 only
c) 1 and 2 only
d) All of the above
Correct Answer: C
Your Answer: Unanswered
Explanation:
Solution (c)

· The new Foreign Trade Policy 2015-20 was unveiled by Commerce & Industry Minister, Mrs. Nirmala
Sitharaman on 1st April 2015. It provides a framework for increasing exports of goods and services as
well as generation of employment and increasing value addition in the country, in keeping with the “Make
in India” vision of Prime Minister. The focus of the new policy is to support both the manufacturing and
services sectors, with a special emphasis on improving the ‘ease of doing business’ .

· The new foreign trade policy aims at increasing exports of goods and services to 900 billion by
2019-20, from $466 billion in 2013-14. It seeks to raise India’s share in world exports from existing 2% to
3.5% in near future.

· The existing reward schemes have been merged into two new schemes as under:

· Merchandise exports from India scheme (MEIS)

· The earlier 5 schemes (Focus Product Scheme, Market Linked Focus Product Scheme, Focus Market
Scheme, Agri. Infrastructure Incentive Scrip, VKGUY ) for rewarding merchandise exports with different
kind of duty scrips with varying conditions have now been replaced by a single scheme called
Merchandise Export from India Scheme (MEIS). The rates of rewards would range from 2% to 5%.
Notified goods exported to notified markets would be rewarded on realized FOB values of exports in free
foreign exchange.

· Service Export from India Scheme (SEIS)

· SEIS have replaced earlier “Served from India Scheme” and aims to encourage export of notified
services from India. SEIS is stated to apply to ‘Service Providers located in India’ instead of ‘Indian
Service Providers’

IASbaba
Web: http://www.iasbaba.com/ Page 3
Email: iasbabailp2016@gmail.com
Exam Title : Test Series 3

Email :

Exam URL : http://test.iasbaba.com

6. Which of the following is not a provision of revised Draft of Indian Financial Code (IFC)?

a) A monetary policy committee headed by the governor of RBI will decide the key policy rates
b) Instead of RBI an independent Debt Management committee will take care of Debt Management.
c) NBFCs will be regulated by RBI
d) A separate consumer agency to set up to protect consumer interest.
Correct Answer: C
Your Answer: Unanswered
Explanation:
Solution (c)

· FSLRC in its report submitted in 2013, recommended a draft financial code to replace the existing
financial sector laws. But this was criticized from various stakeholders and in this background a revised
draft of Indian Financial Code (IFC) was released on 23 July 2015 by the Finance Ministry.

· Provisions of Revised Draft of IFC

· Monetary policy – instead of RBI, a monetary policy committee headed by RBI chairperson will decide
the key policy rates. Out of 7 members, 3 will be from RBI (including RBI chairperson)

· Debt management – instead of RBI, an independent debt management committee will decide it

· NBFCs – instead of RBI, a financial authority will regulate it

· Payment system – Only systematically important payments systems will be regulated by RBI (at
present all PS are regulated bt RBI)

· Consumer protection – setup a separate consumer agency to protect consumer interest (as banks flout
the norms; mis -sell their products). RBI at present has been ignoring this area

· Capital controls : Under FEMA, RBI deals with capital account regulation. The code suggests that
government will "consult" RBI to make rules on capital controls.

· Money markets : At present RBI regulates all these markets. But code recommends separating
regulation of such markets from RBI

· RBI, central govt in their annual report must give a complete disclosure of their performance. This will
bring about much-needed transparency and will reduce information asymmetry that currently plagues the
system

· Establish a Resolution Corporation to carry out resolution of certain types of financial institutions in
distress. At present there is no Resolution Corporation.

IASbaba
Web: http://www.iasbaba.com/ Page 4
Email: iasbabailp2016@gmail.com
Exam Title : Test Series 3

Email :

Exam URL : http://test.iasbaba.com

7. Match the following


1. Angora Goats Jammu & Kashmir
2. Llama South America
3. Patanwadi Gujarat
4. Nali Rajasthan
Select the correct match

a) 1 and 2
b) 2, 3 and 4
c) 1, 2 and 3
d) All
Correct Answer: D
Your Answer: Unanswered
Explanation:
Solution (d)

NCERT Science 7 th - Chapter 3

8. Which of the following are correct regarding MUDRA bank?


1. Its aim is to refinance the Micro Finance Institutions (MFIs) which are engaged in lending to MSMEs.
2. It will be responsible for accreditation and rating of MFIs
3. There are three financial stages for MUDRA- Shishu, Kishore and Tarun.
Select the correct option from the following

a) 1 only
b) 1 and 3
c) 2 and 3
d) All of the above
Correct Answer: D
Your Answer: Unanswered
Explanation:

IASbaba
Web: http://www.iasbaba.com/ Page 5
Email: iasbabailp2016@gmail.com
Exam Title : Test Series 3

Email :

Exam URL : http://test.iasbaba.com

Solution (d)

· MUDRA stands for Micro Units Development Refinance Agency (MUDRA). On 8th April 2015
government launched MUDRA bank.

· Aim – To refinance microfinance Institutions (MFIs) engaged in lending to MSME’s engaged in


manufacturing, trading and service activities.

· How will it achieve the objective –

· Laying down policy guidelines for MFIs

· Registration of MFIs

· Accreditation, rating of MFI

· Promoting technology

· Running a credit guarantee scheme for providing guarantees to loans

Other Facts –

· It will be the part of Small Industries Development Bank of India (SIDBI)

· Pradhan Mantri MUDRA Yojana will be the main support system for MUDRA Bank.

· Started with an initial corpus of Rs. 20,000 crore .

· There are 3 financing stages for MUDRA Bank known as

· Shishu : for start-ups. They are eligible for loan of Rs. 50,000.

· Kishore : entrepreneur will be eligible for a loan ranging from Rs 50,000 to Rs 5 lakh

· Tarun : maximum loan up to Rs. 10 lakh can be provided under this stage.

9. Consider the following regarding ‘The Benami Transactions (Prohibition) Amendment Bill-2015. The bill
seeks to-
1. Amend the definition of Benami transactions.
2. Establish adjudicating authorities and an Appellate Tribunal to deal with benami transactions
3. Specify the penalty for entering into benami transactions
Select the correct option
a) All of the above
b) 1 and 2
c) 2 and 3

IASbaba
Web: http://www.iasbaba.com/ Page 6
Email: iasbabailp2016@gmail.com
Exam Title : Test Series 3

Email :

Exam URL : http://test.iasbaba.com

d) 1 and 3
Correct Answer: A
Your Answer: Unanswered
Explanation:

IASbaba
Web: http://www.iasbaba.com/ Page 7
Email: iasbabailp2016@gmail.com
Exam Title : Test Series 3

Email :

Exam URL : http://test.iasbaba.com

Solution (a)

· The Benami Transactions (Prohibition) Amendment Bill, 2015 was introduced in Lok Sabha on May 13, 2015 by the Minister of
Finance Mr. Arun Jaitley . The Bill seeks to amend the Benami Transactions Act, 1988. The Act prohibits benami transactions
and provides for confiscating benami properties.

Provisions

· The Bill seeks to: ( i ) amend the definition of benami transactions, (ii) establish adjudicating authorities and an Appellate
Tribunal to deal with benami transactions, and (iii) specify the penalty for entering into benami transactions.

· The Act defines a benami transaction as a transaction where a property is held by or transferred to a person, but has been
provided for or paid by another person. The Bill amends this definition to add other transactions which qualify as benami , such
as property transactions where: the transaction is made in a fictitious name, the owner is not aware of denies knowledge of the
ownership of the property, or the person providing the consideration for the property is not traceable.

· The Bill also specifies certain cases will be exempt from the definition of a benami transaction. These include cases when a
property is held by: ( i ) a member of a Hindu undivided family, and is being held for his or another family member’s benefit, and
has been provided for or paid off from sources of income of that family; (ii) a person in a fiduciary capacity; (iii) a person in the
name of his spouse or child, and the property has been paid for from the person’s income; and The Bill defines benamidar as
the person in whose name the benami property is held or transferred, and a beneficial owner as the person for whose benefit
the property is being held by the benamidar .

· Under the Act, an Authority to acquire benami properties was to be established by the Rules. The Bill seeks to establish four
authorities to conduct inquiries or investigations regarding benami transactions: ( i ) Initiating Officer, (ii) Approving Authority, (iii)
Administrator and ( iv ) Adjudicating Authority.

· If an Initiating Officer believes that a person is a benamidar , he may issue a notice to that person.

· The Initiating Officer may hold the property for 90 days from the date of issue of the notice, subject to permission from the
Approving Authority. At the end of the notice period, the Initiating Officer may pass an order to continue the holding of the
property.

· If an order is passed to continue holding the property, the Initiating Officer will refer the case to the Adjudicating Authority.

· Based on an order to confiscate the benami property, the Administrator will receive and manage the property in a manner and
subject to conditions as prescribed.

· The Bill also seeks to establish an Appellate Tribunal to hear appeals against any orders passed by the Adjudicating Authority.
Appeals against orders of the Appellate Tribunal will lie to the high court.

Penalty

· Under the Act, the penalty for entering into benami transactions is imprisonment up to three years, or a fine, or both.

· The Bill seeks to change this penalty to rigorous imprisonment of one year up to seven years, and a fine which may extend to
25% of the fair market value of the benami property.

· The Bill also specifies the penalty for providing false information to be rigorous imprisonment of six months up to five years,
and a fine, which may extend to 10% of the fair market value of the benami property.

· Certain sessions courts would be designated as Special Courts for trying any offences which are punishable under the Bill.

IASbaba
Web: http://www.iasbaba.com/ Page 8
Email: iasbabailp2016@gmail.com
Exam Title : Test Series 3

Email :

Exam URL : http://test.iasbaba.com

10. Which of the tax is not a form of direct tax?

a) Corporate Income tax


b) Securities Transaction Tax
c) Value added tax
d) Fringe Benefit Tax
Correct Answer: C
Your Answer: Unanswered
Explanation:
Solution (c)

· A tax that is paid directly by an individual or organization to the imposing entity. A taxpayer pays a
direct tax to a government for different purposes, including real property tax, personal property tax,
income tax or taxes on assets. Direct taxes are different from indirect taxes, where the tax is levied on
one entity, such as a seller, and paid by another, such a sales tax paid by the buyer in a retail setting. A
direct tax cannot be shifted to another individual or entity. The individual or organization upon which the
tax is levied is responsible for the fulfilment of the tax payment. Indirect taxes, on the other hand, can be
shifted from one taxpayer to another.

· Direct taxes in India include corporate tax, Personal Income Tax, Securities transaction tax, banking
Cash Transaction tax and Fringe Benefit tax.

· Indirect Tax is a tax that increases the price of a good so that consumers are actually paying the tax by
paying more for the products. The Ministry of Finance (Department of Revenue) through the Central
Board of Excise and Customs (CBEC), an apex indirect tax authority, implements and administers excise
(central excise), customs and service tax laws.

· It includes service tax, excise, VAT etc.

11. Consider the following statements with respect to indigenisation of navy


1. INS Kolkata is the first guided missile destroyers indigenously built and operated by Indian Navy
2. So far INS Kochi is the largest indigenized warship built by India
Select the correct answer using the code given below

a) 1 only
b) 2 only
c) Both 1 and 2
d) None of the above
Correct Answer: C
Your Answer: Unanswered
Explanation:

IASbaba
Web: http://www.iasbaba.com/ Page 9
Email: iasbabailp2016@gmail.com
Exam Title : Test Series 3

Email :

Exam URL : http://test.iasbaba.com

Solution (c)

· INS Kolkata was commissioned in August 2014 and weights around 6800 tonnes.

· It was the first ever destroyer built indigenously by mazagon ship dock Mumbai

· The ship is already in operation by Indian navy

· INS Kochi is the largest destroyer warship with 7500 ton capacity and it was commissioned by Indian
navy in Sep 2015

Features of INS Kochi

· This destroyer is capable of launching long range surface to surface Brahmos missile

· It is the second in the Indian Navy, next to INS Kolkata, to have Multi-Function Surveillance and Threat
Alert Radar( MF-STAR) to provide target data to Long Range Surface to Air Missile system (LR-SAM).

· It is the second ship of the Kolkata-class (Project 15A) Guided Missile Destroyers.

· Source- https://en.wikipedia.org/wiki/INS_Kochi_(D64)

12. Recently a report entitled leveraging urbanisation in south Asia was released by an international bank.
Which of the following bank released the above report?

a) World bank
b) Asian development bank
c) National development bank
d) Asian investment infrastructure bank
Correct Answer: A
Your Answer: Unanswered
Explanation:

IASbaba
Web: http://www.iasbaba.com/ Page 10
Email: iasbabailp2016@gmail.com
Exam Title : Test Series 3

Email :

Exam URL : http://test.iasbaba.com

Solution (a)

World Bank released the report

Report with respect to India

· 13.7 percent of urban population or 1 in 7 in urban areas live below the poverty line in the country
compared to 13.1 percent in Pakistan.

· Among the very poorest, under 5 mortality rate is higher in urban than in rural settings.

· Delhi’s air pollution (PM 2.5 concentration) is almost three times higher than in Beijing giving it the
dubious distinction of being the world’s most polluted city.

· For the 12 largest Indian cities the proportion of built-up area outside a city’s official boundaries
exceeds that within its boundaries.

· Governments at all levels should conduct risk assessments from potential natural hazards especially in
cities that lie along the Himalayan range that are at risk of earthquakes and heavy inland flooding.

· Source- http://www.worldbank.org/en/region/sar/publication/urbanization-south-asia-cities

13. Consider the following statement with respect to recently launched satellite “ASTROSAT”
1. ASTROSAT is India’s first dedicated multi wavelength space observatory used for astronomy research
2. It was launched using GSLV launch vehicle
3. USA and Russia are the only two countries that have their own space observatories
Select the correct answer using the code given below

a) 1 and 2 only
b) 3 only
c) 1 only
d) 2 and 3 only
Correct Answer: C
Your Answer: Unanswered
Explanation:

IASbaba
Web: http://www.iasbaba.com/ Page 11
Email: iasbabailp2016@gmail.com
Exam Title : Test Series 3

Email :

Exam URL : http://test.iasbaba.com

Solution (c)

· Indian Space Research Orgnaisation (ISRO) launched India’s first dedicated multi wavelength satellite
for astronomy research, named ASTROSAT.

· Satellite was launched from the first launch pad of Satish Dhawan Space Centre at Sriharikota , Andhra
Pradesh at 10 am by Polar Satellite Launch Vehicle (PSLV-C30)

· US, Japan, Russia and Europe already have their own satellite observatory

Why the launch is unique?

· Astrosat is India’s major scientific mission after the highly acclaimed Chandrayaan -I and Mangalyaan
and this attempt of ISRO will help it to set up an observatory in space, a place from where it can study
cosmological phenomena.

· This launch and its success makes India a member of elite group of nations having their own space
observatory after the US, Japan, Russia and Europe.

· This is the third time that an Indian rocket carried seven satellites in a single mission. In 2008, ISRO
launched 10 satellites in one go including India's Cartosate-2A satellite.

14. Which of the following states given below doesn’t enjoy special status according to Indian constitution?

a) Assam
b) Bihar
c) Himachal Pradesh
d) Uttarkhand
Correct Answer: B
Your Answer: Unanswered
Explanation:
Solution (b)

· At present there are 11 States that enjoy Special Category Status they are: Arunachal Pradesh, Assam,
Himachal Pradesh, Jammu and Kashmir, Manipur, Meghalaya, Mizoram, Nagaland, Sikkim, Tripura and
Uttarakhand .

· Source- Lakshmikanth

IASbaba
Web: http://www.iasbaba.com/ Page 12
Email: iasbabailp2016@gmail.com
Exam Title : Test Series 3

Email :

Exam URL : http://test.iasbaba.com

15. Which of the following statements with respect to Planet Mars is correct?
1. Mars is referred to as the "Red Planet" because of the presence of excess amount of Lithium and
Magnesium
2. Mars moons are called phobos, Titan and Deimos
Select the correct answer using the code given below

a) 1 only
b) 2 only
c) Both 1 and 2
d) None of the above
Correct Answer: D
Your Answer: Unanswered
Explanation:
Solution (d)

· The presence of red colour of mars is due to presence of Iron oxide

· Mars has only two moons phobos and deimos

· Titan is the moon of Saturn

· (Rationale-1 year of mars mission)

16. A democratic polity is based on the doctrine of popular sovereignty. With regard to this principle,
consider the following
1. Plebiscite
2. Referendum
3. Recall
4. Initiative
Which of the above is/are used in direct democracy?

a) 2, 3 and 4
b) 1 and 2
c) 1, 2 and 3
d) All of the above
Correct Answer: D
Your Answer: Unanswered
Explanation:

IASbaba
Web: http://www.iasbaba.com/ Page 13
Email: iasbabailp2016@gmail.com
Exam Title : Test Series 3

Email :

Exam URL : http://test.iasbaba.com

Solutions (d)

· Democracy is of two types—direct and indirect. In direct democracy, the people exercise their supreme
power directly as is the case in Switzerland. There are four devices of direct democracy, namely,
Referendum, Initiative, Recall and Plebiscite.

· In indirect democracy, on the other hand, the representatives elected by the people exercise the
supreme power and thus carry on the government and make the laws. This type of democracy, also
known as representative democracy, is of two kinds—parliamentary and presidential.

· Source : Lakshmikanth Chapter 4 ‘Preamble of the Constitution’

17. The branchy part of a tree above the stem is known as

a) Canopy
b) Crown
c) Sapling
d) Understorey
Correct Answer: B
Your Answer: Unanswered
Explanation:
Solution (b)

NCERT Science- Class 7- Chapter 17

18. Which of the following state was first reorganized based on linguistic basis?

a) Karnataka
b) Maharastra
c) Tamil nadu
d) Andra Pradesh
Correct Answer: D
Your Answer: Unanswered
Explanation:
Solution (d)

· The government of India was forced to create the first linguistic state known as Andhra Pradesh by
separating telugu speaking people from madras state

· This was done after a prolonged popular agitation and death of Potti sriramulu 56 day hunger strike

IASbaba
Web: http://www.iasbaba.com/ Page 14
Email: iasbabailp2016@gmail.com
Exam Title : Test Series 3

Email :

Exam URL : http://test.iasbaba.com

19. The first schedule of the Indian Constitution does not pertains to which one of the following
1. Names of States
2. Names of Union territories
3. Name of the people residing inside the territory of the India
4. Territory that may be acquired by the GOI at any time
Select the correct answer using the code given below:

a) 1 and 4
b) 3 and 2
c) 3 only
d) 1 and 2 only
Correct Answer: C
Your Answer: Unanswered
Explanation:
Solution (c)

· The first schedule of the Indian Constitution pertains to Names of the States and their

· Territorial jurisdiction and Names of the Union Territories and their extent. Forms of Oaths

· And Affirmation comes under Third Schedule.

· Source : Laxmikanth Chapter 5 ‘Union and its Territory’

20. The idea of fraternity in Preamble is taken from

a) American revolution
b) French revolution
c) Russian revolution
d) None of these
Correct Answer: B
Your Answer: Unanswered
Explanation:
Solution (b)

· Republic and ideals of liberty, equality and fraternity in the preamble were taken from French
constitution

· Source- Lakshmikanth chap 4

IASbaba
Web: http://www.iasbaba.com/ Page 15
Email: iasbabailp2016@gmail.com
Exam Title : Test Series 3

Email :

Exam URL : http://test.iasbaba.com

21. According to the Constitution of India, Article 3 authorises the Parliament to reorganise the States. With
this authority, the Parliament can
1. Alter the name of any state
2. Diminish the area of any state
3. Alter the boundaries of any state
4. Increase the area of any state
Select the correct answer using the code given below:

a) 1, 3 and 4
b) 1 and 3
c) 2, 3 and 4
d) All the above
Correct Answer: D
Your Answer: Unanswered
Explanation:
Solution (d)

· Article 3 authorises the Parliament to:

· Form a new state by separation of territory from any state or by uniting two or more states or parts of
states or by uniting any territory to a part of any state,

· Increase the area of any state,

· Diminish the area of any state,

· Alter the boundaries of any state, and

· Alter the name of any state.

· Source : Laxmikanth Chapter 5 ‘Union and its Territory’

22. Which of the following are the sources of Human Capital Formation?
1. Expenditure on education
2. Expenditure on Health
3. Expenditure on Migration
4. Expenditure incurred on acquiring information regarding labour market.
Select the correct option from the following:
a) 1 and 2
b) 3 and 4
c) 1,2 and 4
d) All of the above

IASbaba
Web: http://www.iasbaba.com/ Page 16
Email: iasbabailp2016@gmail.com
Exam Title : Test Series 3

Email :

Exam URL : http://test.iasbaba.com

Correct Answer: D
Your Answer: Unanswered
Explanation:
Solution (d)

· NCERT Indian Economic Development Ch-5

· Investment in education is considered as one of the main sources of human capital. There are several
other sources as well. Investments in health, on the job training, migration and information are the other
sources of human capital formation.

· Spending on education by individuals is similar to spending on capital goods by companies with the
objective of increasing future profits over a period of time. Likewise, individuals invest in education with
the objective of increasing their future income. Preventive medicine (vaccination), curative medicine
(medical intervention during illness), social medicine (spread of health literacy) and provision of clean
drinking water and good sanitation are the various forms of health expenditures. Health expenditure
directly increases the supply of healthy labour force and is, thus, a source of human capital formation.
Unemployment is the reason for the rural-urban migration in India. Technically qualified persons, like
engineers and doctors, migrate to other countries because of higher salaries that they may get in such
countries. Migration in both these cases involves cost of transport, higher cost of living in the migrated
places and psychic costs of living in a strange socio-cultural setup. The enhanced earnings in the new
place outweigh the costs of migration; hence, expenditure on migration is also a source of human capital
formation. People spend to acquire information relating to the labour market and other markets like
education and health. This information is necessary to make decisions regarding investments in human
capital as well as for efficient utilisation of the acquired human capital stock.

· Expenditure incurred for acquiring information relating to the labour market and other markets is also a
source of human capital formation

23. Which of the following is not a provision of Model APMC Act?

a) It is not compulsory for the farmer to bring his produce to APMC Mandi
b) Exporters, graders, packers etc can buy directly from the farmer.
c) It does not include Contract farming.
d) Prohibits commission agents in any transaction.
Correct Answer: C
Your Answer: Unanswered
Explanation:

IASbaba
Web: http://www.iasbaba.com/ Page 17
Email: iasbabailp2016@gmail.com
Exam Title : Test Series 3

Email :

Exam URL : http://test.iasbaba.com

Solution (c)

Provisions for Model APMC Act

· Farmer doesn’t need to bring his produce to APMC Mandi . He can directly sell it to whomever he
wants. (Although, if he doesn’t bring his produce to Mandi , then he can’t run for election in that APMC
marketing committee.)

· Farmers Processors, exporters, graders, packers, etc. can buy agricultural produce directly from
farmers. It Permits Private market yards, Direct Purchase Centers , farmers’ market for doing trade in
agriculture produce. i.e Monopoly of Mandi’s is destroyed

· A separate Chapter to regulate and promote contract-farming arrangements in the country. A Dispute
resolution mechanism has also been made for contract farming.

· Prohibits commission agents in any transaction.

· Establish State Agricultural Produce Marketing Standards Bureau for Grading, Standardization and
Quality Certification of agricultural produce.

24. Consider the following statements:


1. MSP is the minimum price at which the government promises to buy the crop from the farmers.
2. MSP protects the farmer in case of a bumper crop.
3. The price support scheme for oil seeds and pulses is implemented through National Agriculture
Cooperative Marketing Federation of India Ltd.(NAFED)
Select the correct statement:

a) 1 only
b) 2 only
c) 1 and 2
d) All of the above
Correct Answer: D
Your Answer: Unanswered
Explanation:

IASbaba
Web: http://www.iasbaba.com/ Page 18
Email: iasbabailp2016@gmail.com
Exam Title : Test Series 3

Email :

Exam URL : http://test.iasbaba.com

Solution (d)

· Minimum Support Price (MSP) is a form of market intervention by the Government of India to insure
agricultural producers against any sharp fall in farm prices. The minimum support prices are announced
by the Government of India at the beginning of the sowing season for certain crops on the basis of the
recommendations of the Commission for Agricultural Costs and Prices (CACP). MSP is price fixed by
Government of India to protect the producer - farmers - against excessive fall in price during bumper
production years. The minimum support prices are a guarantee price for their produce from the
Government. The major objectives are to support the farmers from distress sales and to procure food
grains for public distribution. In case the market price for the commodity falls below the announced
minimum price due to bumper production and glut in the market, government agencies purchase the
entire quantity offered by the farmers at the announced minimum price.

· If there is a fall in the prices of the crops, after a bumper harvest, the government purchases at the
MSP and this is the reason that the priced cannot go below MSP. So this directly helps the farmers. The
Department of Agriculture and Cooperation implements the Price Support Scheme for Oil Seeds and
Pulses through the National Agricultural Cooperative Marketing Federation of India Ltd. (NAFED).
NAFED is the nodal procurement agency for Oilseeds and pulses, apart from the Cotton Corporation of
India. So, when the prices of oilseeds, pulses and cotton fall below MSP, NAFED purchases them from
the farmers.

25. India’s population growth rate, although high on an absolute scale, has slowed down in past two
decades. Which of the following reasons cannot be attributed to it?

a) Increase in death rate due to TB, dengue, and other infectious diseases.
b) Increase in female literacy rate.
c) Successful family planning schemes in rural areas.
d) Increase in average wedding age.
Correct Answer: A
Your Answer: Unanswered
Explanation:

IASbaba
Web: http://www.iasbaba.com/ Page 19
Email: iasbabailp2016@gmail.com
Exam Title : Test Series 3

Email :

Exam URL : http://test.iasbaba.com

Solution (a)

· Rate of change of population depends broadly on two factors: The natural growth rate. i.e . the
difference between fertility rate and the death rate.

· Polulation explosion takes place when fertility rate is high and death rate is low.

· With improvement in health sector the death rate has gone down. So the first option is incorrect.

· Increase in female literacy has increased their awareness about contraceptives, age gap between
children, and family planning.

· Many family planning measures and awareness programs have been very successful in rural areas in
curbing population growth.

26. Consider the following statements:


1. Gross Domestic Product (GDP) includes the total value of all final goods and services produced in a
country in a year.
2. Intermediate products are included in GDP.
3. Gross National Product (GNP) is the summation of GDP and Net Factor Income from Abroad (NFIA).
Select the correct option from the following:

a) 1 Only
b) 1 and 2
c) 1 and 3
d) All of the above.
Correct Answer: C
Your Answer: Unanswered
Explanation:

IASbaba
Web: http://www.iasbaba.com/ Page 20
Email: iasbabailp2016@gmail.com
Exam Title : Test Series 3

Email :

Exam URL : http://test.iasbaba.com

Solution (c)

· GDP : It is the money value of all final goods and services produced in the domestic territory of the
country during an accounting year.

· Intermediary goods like raw material etc. are not included to avoid double counting.

· GNP: It is the money value of all final goods and services produced by the normal residents of a
country during an accounting year.

· Normal Residents: refers to the individuals who usually resides in a country and whose long term
interest lies in the country. For income tax purpose, a person staying for more than 182 days in an
accounting year.

· Net Factor Income Abroad = Income earned by normal residents from abroad – income earned by non
residents from the domestic territory of the country.

27. Consider the following statements:


Assertion (A): Article 1 describes India, as a ‘Union of States’ rather than ‘Federation of
States’
Reason (R): Indian Federation is not the result of an agreement among the States
Select the correct answer using the code given below:

a) Both A and R are True and R is the correct explanation of A


b) Both A and R are True but R is not the correct explanation of A
c) A is True and R is False
d) A is False and R is True
Correct Answer: A
Your Answer: Unanswered
Explanation:
Solution (a)

· Article 1 describes India, that is, Bharat as a ‘Union of States’ rather than a ‘Federation of States’. This
provision deals with two things: one, name of the country, and two, type of polity. According to Dr B R
Ambedkar , the phrase ‘Union of States’ has been preferred to ‘Federation of States’ for two reasons

· The Indian Federation is not the result of an agreement among the states like the American Federation;

· States have no right to secede from the federation.

· Source : Laxmikanth Chapter 5 ‘Union and its Territory’

IASbaba
Web: http://www.iasbaba.com/ Page 21
Email: iasbabailp2016@gmail.com
Exam Title : Test Series 3

Email :

Exam URL : http://test.iasbaba.com

28. The commitment to equality before law as fundamental rights was declared in

a) Karachi session, 1931


b) Lahore session, 1909
c) Calcutta session, 1929
d) Bombay session, 1934
Correct Answer: A
Your Answer: Unanswered
Explanation:
Solution (a)

· Source: NCERT Class 12 Indian society, Chapter 5

29. In India, the Citizenship Act of 1955 prescribes which of the following ways to acquire citizenship
1. By naturalisation
2. By descent
3. By incorporation of territory
4. By registration
Select the correct answer using the code given below:

a) 1 and 4
b) 1, 2 and 3
c) 1, 2 and 4
d) All of the above
Correct Answer: D
Your Answer: Unanswered
Explanation:
Solution (d)

· The Citizenship Act (1955) provides for acquisition and loss of citizenship after the commencement of
the Constitution The Citizenship Act of 1955 prescribes five ways of acquiring citizenship - birth, descent,
registration, naturalisation and incorporation of territory

· Originally, the Citizenship Act (1955) also provided for the Commonwealth Citizenship. But, this
provision was repealed by the Citizenship (Amendment) Act, 2003.

· Source : Laxmikanth Chapter 6 ‘Citizenship’

30. Compact Disc exhibit rainbow like phenomenon when seen under sunlight because of
a) Reflection and Refraction
b) Refraction and Transmission

IASbaba
Web: http://www.iasbaba.com/ Page 22
Email: iasbabailp2016@gmail.com
Exam Title : Test Series 3

Email :

Exam URL : http://test.iasbaba.com

c) Reflection, Refraction, Transmission and Diffraction


d) Diffraction and Transmission
Correct Answer: C
Your Answer: Unanswered
Explanation:
Solution (c)

Hint- NCERT Science- 7 th Chapter- 15

31. Which among following committee most strongly advocated organisation of state on linguistic
homogeneity?

a) S K Dhar
b) JVP committee
c) Pottu Sriramulu
d) Fazl Ali
Correct Answer: D
Your Answer: Unanswered
Explanation:
Solutions (d)

· Source : Laxmikanth Chapter 5 ‘Union and its Territory’

32. Select the correct statements with respect to citizenship is concerned

a) Children of foreign diplomat, born in India, are given dual citizenship so long as the diplomat
continues to be in service in India
b) IFS officer's son if born in India cannot get citizenship in the country where his parent is serving as
Indian diplomat
c) Both
d) None
Correct Answer: C
Your Answer: Unanswered
Explanation:
Solution (c)

· The matter was in news recently

· Source:
http://www.thehindu.com/news/national/no-indian-passports-to-khobragades-kids-mha/article7452486.ece

IASbaba
Web: http://www.iasbaba.com/ Page 23
Email: iasbabailp2016@gmail.com
Exam Title : Test Series 3

Email :

Exam URL : http://test.iasbaba.com

33. When a switch is in OFF position-


1. Circuit starting from the positive terminal of the cell stops at the switch.
2. Circuit is open.
3. No current flows through it.
4. Current flows after some time.
Choose the combination of correct answer from the following.

a) all are correct


b) 2 and 3 are correct
c) only 4 is correct
d) only 1, 2 and 3 are correct
Correct Answer: B
Your Answer: Unanswered
Explanation:
Solution (b)

NCERT Science 7 th - Chapter 14

34. Recently Nangchen Stupa in China was in news for which of the following reasons
1. It is the place where Huen Tsang died
2. It is one of the places on earth where Buddhas relics are found
3. Fa sein delivered his first sermon to people of china after visiting from India
4. This Stupa was constructed by Emperor Ashoka
Select the correct answer using the code given below

a) 1 and 3 only
b) 2 and 4 only
c) 4 only
d) 3 and 4 only
Correct Answer: B
Your Answer: Unanswered
Explanation:

IASbaba
Web: http://www.iasbaba.com/ Page 24
Email: iasbabailp2016@gmail.com
Exam Title : Test Series 3

Email :

Exam URL : http://test.iasbaba.com

Solution (b)

· The Nangchen stupa , was first discovered in the Tibetan region, is one of the 19 stupas containing
Lord Buddha's relics that were sent by Emperor Ashoka to China.

· It was constructed by Ashoka

· These stupas are a symbol of advent of Buddhism from India to China .

· http://www.nangchen.org/index.php?option=com_content&view=article&id=108&Itemid=504&lang=en

35. Which of the following statements with respect to Diclofenac is incorrect?


1. Diclofenac is used to treat human beings
2. Diclonefac is an anti inflammatory drug used to treat Cattles
Select the correct answer using the code given below

a) 1 only
b) 2 only
c) Both 1 and 2
d) None of the above
e)
f)
Correct Answer: D
Your Answer: Unanswered
Explanation:
Solution (d)

· Diclofenac is a nonsteroidal anti-inflammatory drug (NSAID) that is used to treat mild to moderate pain
or signs and symptoms of osteoarthritis or rheumatoid arthritis.

· Apart from human use, the drug is also effective in treating cattle.

· https://en.wikipedia.org/wiki/Diclofenac

36. Recently India, Cambodia signed two MoUs on Tourism & Mekong Ganga Cooperation Initiative. Which
of the following country capital is not involved in Mekong ganga Initiative

a) Bangkok
b) Kaulalumpur
c) Hanoi
d) Nay Pyi Daw
Correct Answer: B

IASbaba
Web: http://www.iasbaba.com/ Page 25
Email: iasbabailp2016@gmail.com
Exam Title : Test Series 3

Email :

Exam URL : http://test.iasbaba.com

Your Answer: Unanswered


Explanation:
Solution (b)

· Malaysia ( Kaulalumpur ) is not involved in Mekong Ganga initiative

· https://en.wikipedia.org/wiki/Mekong–Ganga_Cooperation

· In recent Current Affairs

37. Recently world’s longest continental volcanic chain was discovered in which of the following countries

a) Japan
b) Indonesia
c) Phillipines
d) Australia
Correct Answer: D
Your Answer: Unanswered
Explanation:
Solution (d)

· Australia

· The 1240-mile-long (2000 kilometres) chain of fire spanned most of eastern Australia, from
Hillsborough in the north, where rainforest meets the Great Barrier Reef, to the island of Tasmania in the
south. This track is three times the length of famous Yellowstone hotspot track on the North American
continent.

·
http://www.theguardian.com/world/2015/sep/15/worlds-longest-continental-volcano-chain-discovered-in-australia

38. Consider the reaction


1. Glucose -------> A + water + energy (presence of oxygen)
2. Glucose -------> A + water + energy (absence of oxygen)
Select the correct answer

a) 1-A is Alcohol & 2-A is Carbon Dioxide


b) 1-A is Carbon Dioxide & 2-A is Alcohol
c) 1-A is Carbon Dioxide & 2-A is Carbon Monoxide
d) 1-A is Alcohol & 2-A is Carbon Monoxide

IASbaba
Web: http://www.iasbaba.com/ Page 26
Email: iasbabailp2016@gmail.com
Exam Title : Test Series 3

Email :

Exam URL : http://test.iasbaba.com

Correct Answer: B
Your Answer: Unanswered
Explanation:
Solution (b)

In the cell, the food (glucose) is broken down into carbon dioxide and water using oxygen. When
breakdown of glucose occurs with the use of oxygen it is called aerobic respiration. Food can also be
broken down, without using oxygen. This is called anaerobic respiration. Breakdown of food releases
energy.

Glucose -------> CO2 + water + energy (presence of oxygen)

Glucose -------> Alcohol + water + energy (absence of oxygen)

NCERT Science 7 th - Chapter 1

39. Which of the following is/are the principal feature(s) of Government of India Act, 1919?
1. It extended principle of communal representation by providing separate electorates for both Sikhs and
Christians.
2. Introduced provincial autonomy
3. Introduced bicameralism in India
Choose the correct answer from the codes given below

a) 1 and 2
b) 2 and 3
c) 1 and 3
d) 1, 2 and 3
Correct Answer: C
Your Answer: Unanswered
Explanation:

IASbaba
Web: http://www.iasbaba.com/ Page 27
Email: iasbabailp2016@gmail.com
Exam Title : Test Series 3

Email :

Exam URL : http://test.iasbaba.com

Solution (c)

Important provisions of GOI act 1919

· It introduced diarchy in provinces by dividing provincial subjects into transferred and reserved subjects.

· It introduced for first time bicameralism and direct elections in the country.

· It extended principle of communal representation to Sikhs, Indian Christians, Anglo Indians and
Europeans.

· It created a new office of high commissioner for India in London.

· It provided for establishment of public service commission.

· It separated for first time provincial budget from central budget.

· Provincial autonomy was introduced by GOI Act 1935.

40. The constituent assembly


1. Was formed as a part of Indian independence act 1947.
2. The assembly consisted of elected members only.
3. Princely states were not given representation in the constituent assembly.
Choose the correct answer from the codes given below

a) None
b) 2 and 3
c) 1 and 3
d) 1 Only
Correct Answer: A
Your Answer: Unanswered
Explanation:

IASbaba
Web: http://www.iasbaba.com/ Page 28
Email: iasbabailp2016@gmail.com
Exam Title : Test Series 3

Email :

Exam URL : http://test.iasbaba.com

Solution (a)

· Constituent assembly was formed as a part of cabinet mission plan 1946.

· Features of constituent assembly

· Its main function was to prepare constitution.

· Total strength was 389 out of which 296 seats were allocated to British India and 93 seats to princely
states.

· From British India members were elected from provinces however from princely states the members
were nominated by head of princely states.

41. The ‘Objective resolution’ passed by Jawaharlal Nehru in the pre-independence era is of historical
importance. Which of the following is correct with regard to the objective resolution?
1. It provided for adequate safeguards for minorities, backward and tribal areas.
2. To proclaim India as an independent sovereign socialist secular republic.
3. Its modified version forms the preamble in the present constitution.
Select the correct answer from the codes given below

a) 1 and 2
b) 2 and 3
c) 1 and 3
d) 1, 2 and 3
Correct Answer: C
Your Answer: Unanswered
Explanation:
Solution (c)

Objective resolution provisions

· It resolves to proclaim India as independent sovereign republic. Socialist and secular was added as a
part of 42nd constitutional amendment 1976.

· India shall be a union of British Indian territories and princely states.

· Power and authority of government will be derived from people.

· Justice, equality and freedom will be guaranteed and secured to all citizens.

· Adequate safeguards shall be provided for minorities, backward and tribal areas.

· The modified version forms the preamble of present constitution.

IASbaba
Web: http://www.iasbaba.com/ Page 29
Email: iasbabailp2016@gmail.com
Exam Title : Test Series 3

Email :

Exam URL : http://test.iasbaba.com

42. With reference to Charter act 1833, consider the following statements
1. It designated Governor General of Bengal as Viceroy of India.
2. It empowered board of control to supervise and direct all operations of civil and military government.
3. It ended activities of East India company as a commercial body.
Choose the correct answer from the codes given below

a) 3 Only
b) 2 and 3
c) 1 and 3
d) 2 Only
Correct Answer: A
Your Answer: Unanswered
Explanation:
Solution (a)

Provisions of charter act 1833

· It made Governor General of Bengal as Governor General of India. William Bentinck was the first
governor general of India.

· It deprived governor of Bombay and madras of their legislative powers.

· It ended activities of east India company as a commercial body.

· It empowered board of control to supervise and direct all operations of civil and military government-
this is a part of Pitts India Act 1784.

43. Which of the following parts of a plant take part in sexual reproduction?
1. Flower
2. Fruit
3. Seed
4. Branch
Choose the correct answer from below.

a) 1 and 3
b) 1, 2 and 3
c) 3 and 4
d) 2, 3 and 4
Correct Answer: A
Your Answer: Unanswered
Explanation:

IASbaba
Web: http://www.iasbaba.com/ Page 30
Email: iasbabailp2016@gmail.com
Exam Title : Test Series 3

Email :

Exam URL : http://test.iasbaba.com

Solution (a)

NCERT Science 7 th - Chapter 12

44. Which of the following are the features of Parliamentary form of government in India?
1. Dissolution of upper house.
2. Moral responsibility of executive to legislature.
3. Majority party rule.
Choose the correct answer from the codes given below

a) 1 and 2
b) 2 Only
c) 3 Only
d) 2 and 3
Correct Answer: C
Your Answer: Unanswered
Explanation:
Solution (c)

· Features of parliamentary government in India

· Presence of nominal and real executive

· Majority party rule

· Collective responsibility of executive to legislature. Morally responsibility doesn’t exist in Indian


parliament.

· Leadership of prime minister or chief minister.

· Dissolution of lower house and not upper house.

45. Which of the following is/are list of fundamental duties contained in part IVA of Indian constitution
1. To safeguard public property and abjure violence
2. To prevent slaughtering of cows.
3. To pay income tax regularly without any avoidance.
Choose the correct answer from the codes given below

a) 1 Only
b) 1 and 2
c) 1 and 3
d) 1, 2 and 3

IASbaba
Web: http://www.iasbaba.com/ Page 31
Email: iasbabailp2016@gmail.com
Exam Title : Test Series 3

Email :

Exam URL : http://test.iasbaba.com

Correct Answer: A
Your Answer: Unanswered
Explanation:
Solution (a)

List of fundamental duties

· to abide by the constitution and respect its ideal and institutions;

· to cherish and follow the noble ideals which inspired our national struggle for freedom;

· to uphold and protect the sovereignty, unity and integrity of India;

· to defend the country and render national service when called upon to do so;

· to promote harmony and the spirit of common brotherhood amongst all the people of India transcending
religious, linguistic and regional diversities, to renounce practices derogatory to the dignity of women;

· to value and preserve the rich heritage of our composite culture;

· to protect and improve the natural environment including forests, lakes, rivers, and wild-life and to have
compassion for living creatures;

· to develop the scientific temper, humanism and the spirit of inquiry and reform;

· to safeguard public property and to abjure violence;

· to strive towards excellence in all spheres of individual and collective activity, so that the nation
constantly rises to higher levels of endeavor and achievement.

· who is a parent or guardian , to provide opportunities for education to his child, or as the case may be,
ward between the age of six and fourteen years.

46. Which of the following language[s] is/are not in the list of languages contained in the Eighth schedule of
Indian constitution?
1. English
2. Pali
3. Sindhi
Choose the correct answer from the codes given below

a) 2 Only
b) 3 Only
c) 1 and 2
d) 1 Only

IASbaba
Web: http://www.iasbaba.com/ Page 32
Email: iasbabailp2016@gmail.com
Exam Title : Test Series 3

Email :

Exam URL : http://test.iasbaba.com

Correct Answer: C
Your Answer: Unanswered
Explanation:

IASbaba
Web: http://www.iasbaba.com/ Page 33
Email: iasbabailp2016@gmail.com
Exam Title : Test Series 3

Email :

Exam URL : http://test.iasbaba.com

Solution (c)

Eight schedule languages

1. Assamese

2. Bengali

3. Bodo

4. Dogri

5. Gujarati

6. Hindi

7. Kannada

8. Kashmiri

9. Konkani

10. Maithili

11. Malayalam

12. Manipuri

13. Marathi

14. Nepali

15. Oriya

16. Punjabi

17. Sanskrit

18. Santhali

19. Sindhi

20. Tamil

21. Telugu

22. Urdu

IASbaba
Web: http://www.iasbaba.com/ Page 34
Email: iasbabailp2016@gmail.com
Exam Title : Test Series 3

Email :

Exam URL : http://test.iasbaba.com

47. Which of the following statements is/are correct with regard to Schedule 8 of Indian constitution? The list
of languages also includes
1. Classical languages
2. Official language
3. National language
Choose the correct answer from the codes given below

a) 1 and 2 Only
b) 1, 2 and 3
c) 1 and 3
d) 2 Only
Correct Answer: A
Your Answer: Unanswered
Explanation:

IASbaba
Web: http://www.iasbaba.com/ Page 35
Email: iasbabailp2016@gmail.com
Exam Title : Test Series 3

Email :

Exam URL : http://test.iasbaba.com

Solution (a)

Schedule 8 contains the list of languages recognised by Indian constitution.

· Classical language: any language which has

· High antiquity of its early texts/recorded history over a period of 1500–2000 years; a body of ancient literature/texts, which is considered a valuable
heritage by generations of speakers; the literary tradition be original and not borrowed from another speech community; the classical language and
literature being distinct from modern, there may also be a discontinuity between the classical language and its later forms or its offshoots.

· Sanskrit, Kannada, Telugu, Tamil, Oriya, Malayalam are classical languages

Eight schedule languages

1. Assamese

2. Bengali

3. Bodo

4. Dogri

5. Gujarati

6. Hindi (Official language)

7. Kannada

8. Kashmiri

9. Konkani

10. Maithili

11. Malayalam

12. Manipuri

13. Marathi

14. Nepali

15. Oriya

16. Punjabi

17. Sanskrit

18. Santhali

19. Sindhi

20. Tamil

21. Telugu

22. Urdu

· Official languages of India – Hindi and English

· There is no officially declared national language of India

· For further reference you can refer the below link-

· http://www.thehindu.com/news/national/hindi-not-a-national-language-court/article94695.ece

· http://timesofindia.indiatimes.com/india/Theres-no-national-language-in-India-Gujarat-High-Court/articleshow/5496231.cms

IASbaba
Web: http://www.iasbaba.com/ Page 36
Email: iasbabailp2016@gmail.com
Exam Title : Test Series 3

Email :

Exam URL : http://test.iasbaba.com

48. Select the incorrect statement

a) The mirrors used as side mirrors in scooters are Convex mirrors


b) The mirrors used by doctors to examine eyes, ears and nose etc are concave mirrors
c) Magnifying glasses are types of concave mirrors
d) Both (a) and (c)
Correct Answer: C
Your Answer: Unanswered
Explanation:
Solution (c)

Magnifying glass is a type of lens. The other name for convex lens is magnifying GLASS.
It is also called as converging lens. The size of the virtual image of an object seen through this lens is
increased. Concave lens is called diverging lens. The size of the virtual image of an object seen through
this lens is diminished

NCERT Science Class 7 th - Chapter 15

49. The first schedule of the Indian Constitution pertains to


1. Names of States
2. Names of Union territories
3. Citizenship
4. Forms of Oaths and Affirmation
Choose the correct answer using the code given below:

a) 1 and 3
b) 1 and 2
c) 4 only
d) 1 only
Correct Answer: B
Your Answer: Unanswered
Explanation:
Solution (b)

· The first schedule of the Indian Constitution pertains to Names of the States and their territorial
jurisdiction and Names of the Union Territories and their extent. Forms of Oaths and Affirmation come
under Third Schedule.

IASbaba
Web: http://www.iasbaba.com/ Page 37
Email: iasbabailp2016@gmail.com
Exam Title : Test Series 3

Email :

Exam URL : http://test.iasbaba.com

50. Schedule 5 of the Indian constitution incorporates


1. Provisions relating to administration and control of scheduled areas and scheduled tribes
2. Provisions for establishment of district and regional councils.
3. Schedule areas of Assam, Meghalaya, Tripura and Mizoram are governed by this schedule.
Choose the correct answer from the codes given below

a) 1 Only
b) 2 Only
c) 1 and 3
d) 1, 2 and 3
Correct Answer: A
Your Answer: Unanswered
Explanation:
Solution (a)

Fifth schedule:

· It contains provisions relating to administration and control of schedule and tribal areas other than the
areas in Assam, Meghalaya, Tripura and Mizoram.

· Schedule and tribal areas in Assam, Meghalaya, Tripura and Mizoram are governed under sixth
schedule.

· It provides provision for establishment of tribes advisory council.

· Tribes Advisory Council consists of not more than twenty members of whom, as nearly as may be,
three-fourths shall be the representatives of the Scheduled Tribes in the Legislative Assembly of the
State.

· Tribes Advisory Council objective is to advise on such matters pertaining to the welfare and
advancement of the Scheduled Tribes in the State as may be referred to them by the Governor.

· District and Regional councils are part of sixth schedule.

51. consider the following regarding Payment Banks: Select the incorrect option from the above.

a) An individual is allowed to hold a maximum of Rs 100000 balance.


b) They will not issue any debit cards or credit cards.
c) They cannot undertake lending activities.
d) They will only provide payment/remittance services.
Correct Answer: B
Your Answer: Unanswered

IASbaba
Web: http://www.iasbaba.com/ Page 38
Email: iasbabailp2016@gmail.com
Exam Title : Test Series 3

Email :

Exam URL : http://test.iasbaba.com

Explanation:
Solution (b)

· The objectives of setting up of payments banks will be to further financial inclusion by providing ( i )
small savings accounts and (ii) payments/remittance services to migrant labour workforce, low income
households, small businesses, other unorganised sector entities and other users.

· Scope of activities :

· Acceptance of demand deposits. Payments bank will initially be restricted to holding a maximum
balance of Rs. 100,000 per individual customer.

· Issuance of ATM/debit cards. Payments banks, however, cannot issue credit cards.

· Payments and remittance services through various channels.

· BC of another bank, subject to the Reserve Bank guidelines on BCs.

· Distribution of non-risk sharing simple financial products like mutual fund units and insurance products,
etc.

· Deployment of funds :

· The payments bank cannot undertake lending activities.

· Apart from amounts maintained as Cash Reserve Ratio (CRR) with the Reserve Bank on its outside
demand and time liabilities, it will be required to invest minimum 75 per cent of its "demand deposit
balances" in Statutory Liquidity Ratio (SLR) eligible Government securities/treasury bills with maturity up
to one year and hold maximum 25 per cent in current and time/fixed deposits with other scheduled
commercial banks for operational purposes and liquidity management.

52. Consider the following:


1. Cooperative banks are organized under the Cooperative society acts of the state governments.
2. They provided concessional credit to agriculture and rural sector.
3. They usually provided long term loans (3-5 years).
Select the correct option:

a) 1 only
b) 1 and 2
c) 2 and 3
d) All of the above
Correct Answer: B
Your Answer: Unanswered

IASbaba
Web: http://www.iasbaba.com/ Page 39
Email: iasbabailp2016@gmail.com
Exam Title : Test Series 3

Email :

Exam URL : http://test.iasbaba.com

Explanation:
Solution (b)

· Cooperative banks are organized under the Cooperative Societies Act of the State Government.

· The main objective is social welfare by providing concessional loans to agriculture and rural sector.
They usually provide short term loans (1-3 years).

· They are organized in a three tiered structure in most of the states.

53. What do you understand by insider trading?

a) It refers to sale and purchase of shares by leaking price sensitive unpublished information.
b) It refers to a sale of products at factory outlet on cheaper price.
c) It refers to selling of products by one country in another country, under the market price to capture the
market.
d) It refers to direct buying of agricultural product from the farms.
Correct Answer: A
Your Answer: Unanswered
Explanation:
Solution (a)

· Insider trading refers to the sale and purchase of shares (by top management of the company) on the
basis of price sensitive unpublished information. It includes leaking information to brokers etc.

· It is a criminal offence in India according to Companies Act.

54. Consider the following regarding Shram Suvidha Portal: Which of the above is not included in Shram
Suvidha Portal?

a) Unique labour identification number.


b) Filing of self certified single online return by Industries
c) Transparent labour inspection scheme via computerized system.
d) Fixed Government wage rates for skilled and unskilled labour.
Correct Answer: D
Your Answer: Unanswered
Explanation:

IASbaba
Web: http://www.iasbaba.com/ Page 40
Email: iasbabailp2016@gmail.com
Exam Title : Test Series 3

Email :

Exam URL : http://test.iasbaba.com

Solution (d)

· The Ministry of Labour & Employment has developed a unified Web Portal ' Shram Suvidha ', catering
to four major organisations under its aegis: Office of Chief Labour Commissioner (Central); Directorate
General of Mines Safety; Employees' Provident Fund Organization; and Employees' State Insurance
Corporation.

· The portal's main features are: Unique Labour Identification Number (LIN) allotted to Units facilitating
online registration. Filing of self-certified, simplified Single Online Return by industry. Units will only file a
single consolidated Online Return, instead of separate Returns. Amendments to 10 Rules already
undertaken. Transparent Labour Inspection Scheme via computerized system as per risk-based criteria
and uploading of Inspection Reports within 72 hours by Labour Inspectors.

55. Recently a landmark scheme for social security was launched by name Atal Pension Yojna. Which of the
following points are true about the scheme?
1. It will guarantee a monthly pension of Rs 1000-10,000.
2. For every contribution made in pension fund, government will contribute an equal amount, with upper
cap of Rs 1000.
3. The scheme targets the unorganised sector.
Select the correct option

a) All of the above


b) 1 only
c) 1 and 3
d) 2 and 3
Correct Answer: D
Your Answer: Unanswered
Explanation:

IASbaba
Web: http://www.iasbaba.com/ Page 41
Email: iasbabailp2016@gmail.com
Exam Title : Test Series 3

Email :

Exam URL : http://test.iasbaba.com

Solution (d)

· Atal Pension Yojana is a government-backed pension scheme in India targeted at the unorganised
sector. It was originally mentioned in the 2015 Budget speech by Finance Minister Arun Jaitley in
February 2015. It was formally launched by Prime Minister Narendra Modi on 9 May in Kolkata. As of
May 2015, only 11% of India's population has any kind of pension scheme, this scheme aims to increase
the number.

· In Atal Pension Yojana , for every contribution made to the pension fund, the government will contribute
an equal amount to his/her fund. Depending on the contribution made between 18 and 40, at the age of
60 a sum of ■1000, ■2000, ■3000, ■4000, or ■5000 will be paid monthly.

· This scheme will be linked to the bank accounts opened under the Pradhan Mantri Jan Dhan Yojana
scheme and the contributions will be deducted automatically. Most of these accounts had zero balance
initially. The government aims to reduce the number of such zero balance accounts by using this and
related schemes.

56. Which of the following correctly explains disguised unemployment?

a) Where part of the labor force is either left without work or is working in a redundant manner where
worker productivity is essentially zero.
b) Those workers those are highly skilled but working in low paying jobs, workers that are highly skilled
but work in low skill jobs and part-time workers that would prefer to be full-time.
c) When people who are out of work are not counted in official unemployment statistics for a variety of
reasons.
d) When people are without work and actively seeking work.
Correct Answer: A
Your Answer: Unanswered
Explanation:

IASbaba
Web: http://www.iasbaba.com/ Page 42
Email: iasbabailp2016@gmail.com
Exam Title : Test Series 3

Email :

Exam URL : http://test.iasbaba.com

Solution (a)

· Unemployment that does not affect aggregate output. Disguised unemployment exists where part of the
labor force is either left without work or is working in a redundant manner where worker productivity is
essentially zero. An economy demonstrates disguised unemployment where productivity is low and
where too many workers are filling too few jobs

· A measure of employment and labor utilization in the economy that looks at how well the labor force is
being utilized in terms of skills, experience and availability to work. Labor that falls under the
underemployment classification includes those workers that are highly skilled but working in low paying
jobs, workers that are highly skilled but work in low skill jobs and part-time workers that would prefer to
be full-time. This is different from unemployment in that the individual is working but isn't working at their
full capability.

· When people who are out of work are not counted in official unemployment statistics for a variety of
reasons. Those potential workers falling into this category typically include individuals who have given up
looking for employment, those who have taken an early retirement even though they would prefer to still
be employed and those with seasonal or part time employment.

57. Pollination refers to the

a) Transfer of pollen from anther to ovary.


b) Transfer of male gametes from anther to stigma.
c) Transfer of pollen from anther to stigma.
d) Transfer of pollen from anther to ovule.
Correct Answer: C
Your Answer: Unanswered
Explanation:
Solution (c)

The transfer of pollen from the anther to the stigma of a flower is called pollination. If the pollen lands on
the stigma of the same flower it is called self-pollination. When the pollen of a flower lands on the stigma
of another flower of the same plant, or that of a different plant of the same kind, it is called
cross-pollination

NCERT Science 7 th - Chapter 12

IASbaba
Web: http://www.iasbaba.com/ Page 43
Email: iasbabailp2016@gmail.com
Exam Title : Test Series 3

Email :

Exam URL : http://test.iasbaba.com

58. Consider the following regarding National Manufacturing Policy:


1. It aims to create 100 million jobs in manufacturing sector.
2. An increase in share of manufacturing sector in India’s GDP from 16% to 25% till 2022.
3. Increase in growth of manufacturing sector to 12 to 14 percent across the medium term.
Which of the above statements are correct?

a) All of the above


b) 1 and 2
c) 2 and 3
d) 1 and 3
Correct Answer: A
Your Answer: Unanswered
Explanation:
Solution (a)

· The need to raise the global competitiveness of the Indian manufacturing sector is imperative for the
country’s long term-growth. The National Manufacturing Policy is by far the most comprehensive and
significant policy initiative taken by the Government. The policy is the first of its kind for the
manufacturing sector as it addresses areas of regulation, infrastructure, skill development, technology,
availability of finance, exit mechanism and other pertinent factors related to the growth of the sector.

It has following vision:

· An increase in manufacturing sector growth to 12-14% per annum over the medium term.

· An increase in the share of manufacturing in the country’s Gross Domestic Product from 16% to 25%
by 2022.

· To create 100 million additional jobs by 2022 in manufacturing sector.

· Creation of appropriate skill sets among rural migrants and the urban poor for inclusive growth.

· An increase in domestic value addition and technological depth in manufacturing.

· Enhancing the global competitiveness of the Indian manufacturing sector.

· Ensuring sustainability of growth, particularly with regard to environment.

59. The Gini Coefficient is a measure of


a) Ratio of women and men in Indian Parliament
b) Migration rate of Guineas nationals
c) Income inequality

IASbaba
Web: http://www.iasbaba.com/ Page 44
Email: iasbabailp2016@gmail.com
Exam Title : Test Series 3

Email :

Exam URL : http://test.iasbaba.com

d) Ratio of coinage to currency note


Correct Answer: C
Your Answer: Unanswered
Explanation:
Solution (c)

· Gini -coefficient of inequality: This is the most commonly used measure of inequality. The coefficient
varies between 0, which reflects complete equality and 1, which indicates complete inequality (one
person has all the income or consumption, all others have none). Graphically, the Gini coefficient can be
easily represented by the area between the Lorenz curve and the line of equality.

· http://siteresources.worldbank.org/INTPA/Images/429965-1092757560964/fig_1_4.gif

60. “To uphold and protect the sovereignty, unity and integrity of India” has been provided in which of the
following of Indian constitution

a) Preamble
b) Directive principles of state policy
c) Fundamental rights
d) Fundamental duties
Correct Answer: D
Your Answer: Unanswered
Explanation:

IASbaba
Web: http://www.iasbaba.com/ Page 45
Email: iasbabailp2016@gmail.com
Exam Title : Test Series 3

Email :

Exam URL : http://test.iasbaba.com

Solution (d)

Article 51A, Fundamental duties: It shall be the duty of every citizen of India

(a) to abide by the Constitution and respect its ideals and institutions, the national Flag and the National
Anthem;

(b) to cherish and follow the noble ideals which inspired our national struggle for freedom;

(c) to uphold and protect the sovereignty, unity and integrity of India;

(d) to defend the country and render national service when called upon to do so;

(e) to promote harmony and the spirit of common brotherhood amongst all the people of India
transcending religious, linguistic and regional or sectional diversities; to renounce practices derogatory to
the dignity of women;

(f) to value and preserve the rich heritage of our composite culture;

(g) to protect and improve the natural environment including forests, lakes, rivers and wild life, and to
have compassion for living creatures;

(h) to develop the scientific temper, humanism and the spirit of inquiry and reform;

to safeguard public property and to abjure violence;

(j) to strive towards excellence in all spheres of individual and collective activity so that the nation
constantly rises to higher levels of endeavour and achievement

(k )who is a parent or guardian to provide opportunities for education to his child or, as the case may be,
ward between the age of six and fourteen years.

Preamble says fraternity assuring dignity of individual and the unity and integrity of nation.

61. Consider the following


1. Preamble was not part of the Original Constitution, it was added by 42nd Constitutional amendment
Act 1976.
2. The parliament, if need arises, can remove the word socialist and add capitalist, in place of socialist.
Choose the correct answer from the codes given below

a) 1 Only
b) 2 Only
c) Both 1 and 2
d) None

IASbaba
Web: http://www.iasbaba.com/ Page 46
Email: iasbabailp2016@gmail.com
Exam Title : Test Series 3

Email :

Exam URL : http://test.iasbaba.com

Correct Answer: D
Your Answer: Unanswered
Explanation:
Solution (d)

· Preamble was present in Original Constitution; it was the last part to be added to constitution.

· Nothing can be removed from preamble however the preamble can be enriched by adding more words.
Preamble is a part of basic structure.

· So parliament cannot remove socialist, however with the presence of socialist the parliament can add
capitalist.

62. Consider the following


1. The term ‘Economic justice’ is present both in Preamble and Directive Principles of State Policy of
Indian constitution.
2. The ideal of economic justice was taken from French revolution (1786).
Choose the correct answer from the codes given below

a) 1 only
b) 2 only
c) Both 1 and 2
d) None
Correct Answer: A
Your Answer: Unanswered
Explanation:

IASbaba
Web: http://www.iasbaba.com/ Page 47
Email: iasbabailp2016@gmail.com
Exam Title : Test Series 3

Email :

Exam URL : http://test.iasbaba.com

Solution (a)

· Preamble:

· WE, THE PEOPLE OF INDIA, having solemnly resolved to constitute India into a SOVEREIGN
SOCIALIST SECULAR DEMOCRATIC REPUBLIC and to secure to all its citizens:

· JUSTICE, social, economic and political;

· LIBERTY of thought, expression, belief, faith and worship;

· EQUALITY of status and of opportunity;

· and to promote among them all

· FRATERNITY assuring the dignity of the individual and the unity and integrity of the Nation;

· IN OUR CONSTITUENT ASSEMBLY this twenty-sixth day of November, 1949, do HEREBY ADOPT,
ENACT AND GIVE TO OURSELVES THIS CONSTITUTION.

· Directive principles of state policy

· Article 38. State to secure a social order for the promotion of welfare of the people

· The State shall strive to promote the welfare of the people by securing and protecting as effectively as it
may a social order in which justice, social, economic and political, shall inform all the institutions of the
national life.

· Economic justice is present in both preamble and DPSP.

· Ideal of economic justice is taken from Russian revolution.

· Ideas of liberty, equality and fraternity were taken from the French revolution.

63. Consider the following


1. The preamble is a source of power to legislature and it enriches the powers of legislature.
2. Both Fundamental duties and Directive Principles of State Policy cannot be enforced in the courts of
law.
Choose the correct answer from the codes given below

a) 1 only
b) 2 only
c) Both 1 and 2
d) None
Correct Answer: B

IASbaba
Web: http://www.iasbaba.com/ Page 48
Email: iasbabailp2016@gmail.com
Exam Title : Test Series 3

Email :

Exam URL : http://test.iasbaba.com

Your Answer: Unanswered


Explanation:
Solution (b)

· Through Keshavananda Bharti case and LIC of India case, it is established that preamble is a part of
India constitution. And also two things should be noted

· The preamble is neither a source of power to legislature nor a prohibition upon powers of legislature. It
is just a foreword to constitution. Legislature does not derive power from preamble.

· Both Fundamental duties and Directive Principles of State Policy are non justiciable i.e., they cannot be
enforced in the courts of law.

· The Fundamental Rights are not absolute and subject to reasonable restrictions. Further , they are not
sacrosanct and can be curtailed or repealed by the Parliament through a constitutional amendment act.
They can also be suspended during the operation of a National Emergency except the rights guaranteed
by Articles 20 and 21.

64. Consider the following


1. The Constituent Assembly which was constituted in November 1946 was not only entrusted with
framing the Constitution but later carried on legislative functions as well.
2. Basically the Indian Independence Act 1947 was the implementation of Mountbatten Plan.
Choose the correct answer from the codes given below

a) 1 only
b) 2 only
c) Both 1 and 2
d) None
Correct Answer: C
Your Answer: Unanswered
Explanation:

IASbaba
Web: http://www.iasbaba.com/ Page 49
Email: iasbabailp2016@gmail.com
Exam Title : Test Series 3

Email :

Exam URL : http://test.iasbaba.com

Solution (c)

· The Indian Independence Act of 1947 made the following changes in the position of the Assembly:

· The Assembly was made a fully sovereign body, which could frame any Constitution it pleased. The act
empowered the Assembly to abrogate or alter any law made by the British Parliament in relation to India.

· The Assembly also became a legislative body. In other words, two separate functions were assigned to
the Assembly that is, making of a constitution for free India and enacting of ordinary laws for the country.
These two tasks were to be performed on separate days. Thus, the Assembly became the first
Parliament of free India (Dominion Legislature). Whenever the Assembly met as the Constituent body it
was chaired by Dr. Rajendra Prasad and when it met as the legislative body6, it was chaired by G V
Mavlankar . These two functions continued till November 26, 1949, when the task of making the
Constitution was over.

· June 3 Plan was also known as the Mountbatten Plan. The British government proposed a plan
announced on 3 June 1947 that included these principles:

· Principle of Partition of India was accepted by the British Government

· Successor governments would be given dominion status

· Implicit right to secede from the British Commonwealth

· The Indian Independence Act 1947 was the implementation of June 3 Plan (Mountbatten Plan)

65. A democratic polity is based on the doctrine of popular sovereignty. With regard to this principle,
consider the following
1. Plebiscite
2. Referendum
3. Recall
4. Initiative
Which of the above is/are used in direct democracy?

a) 2, 3 and 4
b) 1 and 2
c) 1, 2 and 3
d) All of the above
Correct Answer: D
Your Answer: Unanswered
Explanation:

IASbaba
Web: http://www.iasbaba.com/ Page 50
Email: iasbabailp2016@gmail.com
Exam Title : Test Series 3

Email :

Exam URL : http://test.iasbaba.com

Solution (d)

· Democracy is of two types—direct and indirect. In direct democracy, the people exercise their supreme
power directly as is the case in Switzerland. There are four devices of direct democracy, namely,
Referendum, Initiative, Recall and Plebiscite.

· In indirect democracy, on the other hand, the representatives elected by the people exercise the
supreme power and thus carry on the government and make the laws. This type of democracy, also
known as representative democracy, is of two kinds—parliamentary and presidential.

66. Consider the following statements:


Assertion (A) : January 26 was chosen as the date of commencement of the Constitution
Reason (R) : It was on this day in 1930 that Purna Swaraj day was celebrated
Select the correct answer using the code given below:

a) Both A and R are True and R is the correct explanation of A


b) Both A and R are True but R is not the correct explanation of A
c) A is True and R is False
d) A is False and R is True
Correct Answer: A
Your Answer: Unanswered
Explanation:
Solution (a)

· January 26 was specifically chosen as the ‘date of commencement’ of the Constitution because of its
historical importance. It was on this day in 1930 that Purna Swaraj day was celebrated, following the
resolution of the Lahore Session (December 1929) of the INC.

67. The Indian Constitution is the lengthiest written Constitution and a borrowed one. In this context answer
the following

Features Borrowed Sources


1. Procedure for amendment of the Constitution a)South African Constitution
2. Concurrent List b)Japanese Constitution
3. Federation with a strong centre c)Canadian Constitution
4. Procedure established by Law d)Australian Constitution
Select the correct answer using the code given below:
a) 1-d, 2-a, 3-b, 4-c
b) 1-a, 2-c, 3-d, 4-b
c) 1-d, 2-b, 3-a, 4-c
d) 1-a, 2-d, 3-c, 4-b

IASbaba
Web: http://www.iasbaba.com/ Page 51
Email: iasbabailp2016@gmail.com
Exam Title : Test Series 3

Email :

Exam URL : http://test.iasbaba.com

Correct Answer: D
Your Answer: Unanswered
Explanation:
Solution (d)

· Self Explanatory

68. Consider the following statements


1. The Directive Principles of State Policy was inspired from Irish Constitution
2. Fundamental Rights can be amended by the parliament
Choose the correct answer from the codes given below

a) 1 only
b) 2 only
c) Both 1 and 2
d) None
Correct Answer: C
Your Answer: Unanswered
Explanation:
Solution (c)

· The structural part of the Constitution is, to a large extent, derived from the Government of India Act of
1935. The philosophical part of the Constitution (the Fundamental Rights and the Directive Principles of
State Policy) derive their inspiration from the American and Irish Constitutions respectively. The political
part of the Constitution (the principle of Cabinet Government and the relations between the executive
and the legislature) have been largely drawn from the British Constitution.

· The Fundamental Rights are not absolute and subject to reasonable restrictions. Further, they are not
sacrosanct and can be curtailed or repealed by the Parliament through a constitutional amendment act.
They can also be suspended during the operation of a National Emergency except the rights guaranteed
by Articles 20 and 21.

69. As per Article 368, the Indian Constitution can be amended by


1. Simple majority of the Parliament
2. Special majority of the Parliament
3. Special majority of the Parliament and with the ratification by half of the total states
Select the correct answer using the code given below:
a) 2 and 3 only
b) 1 and 2
c) 1 and 3

IASbaba
Web: http://www.iasbaba.com/ Page 52
Email: iasbabailp2016@gmail.com
Exam Title : Test Series 3

Email :

Exam URL : http://test.iasbaba.com

d) All of the above


Correct Answer: A
Your Answer: Unanswered
Explanation:
Solution (a)

· The Constitution of India is neither rigid nor flexible but a synthesis of both. Article 368 provides for two
types of amendments:

· Some provisions can be amended by a special majority of the Parliament, i.e., a two-third

· majority of the members of each House present and voting, and a majority (that is, more than 50 per
cent), of the total membership of each House.

· Some other provisions can be amended by a special majority of the Parliament and with the ratification
by half of the total states.

· At the same time, some provisions of the Constitution can be amended by a simple majority of the
Parliament in the manner of ordinary legislative process. Notably, these amendments do not come under
Article 368.

70. The Constitution of India provides for a federal system of Government in India. But certain federal
features have a unitary bias. Which of the following features does not provide a unitary shade to the
Indian system?

a) Integrated Judiciary
b) Emergency provisions
c) Appointment of Governor
d) Division of powers
Correct Answer: D
Your Answer: Unanswered
Explanation:
Solution (d)

· Division of Powers is one of the main features of any Federal system of Government. In India, the
powers are divided between the Centre and State interms of Union List, State List and Concurrent List.
Though there is a criticism that, the concurrent powers tend to give a unitary shade to the system, option
(d) is the best choice among the given options.

IASbaba
Web: http://www.iasbaba.com/ Page 53
Email: iasbabailp2016@gmail.com
Exam Title : Test Series 3

Email :

Exam URL : http://test.iasbaba.com

71. Consider the following with regard to Poona Pact, 1932


1. It retained the Hindu joint electorate
2. Reservation was provided for the depressed classes
3. Communal Award was the precursor to the Poona pact
4. It was signed by Mahatma Gandhi and B R Ambedkar
Select the correct answer using the code given below:

a) 1 and 2 only
b) 1, 2 and 4
c) 1, 2 and 3
d) All of the above
Correct Answer: C
Your Answer: Unanswered
Explanation:
Solution (c)

· Communal Award In August 1932, Ramsay MacDonald, the British Prime Minister, announced a
scheme of representation of the minorities, which came to be known as the Communal Award. The
award not only continued separate electorates for the Muslims, Sikhs, Indian Christians, Anglo-Indians
and Europeans but also extended it to the depressed classes (scheduled castes). Gandhiji was
distressed over this extension of the principle of communal representation to the depressed classes and
undertook fast unto death in Yeravada Jail (Poona) to get the award modified. At last, there was an
agreement between between Dr. Babasaheb Ambedkar and Mahatma Gandhi on 24 September 1932 at
Yerwada Central Jail in Pune (now in Maharashtra), India. It was signed by Pt Madan Mohan Malviya
and Dr. B.R. Ambedkar and some Dalit leaders to break the fast unto death undertaken by Gandhi in
Yerwada jail to annul Macdonald Award giving separate electorate to Dalits for electing members of state
legislative assemblies in British India.

· The agreement, known as Poona Pact, retained the Hindu joint electorate and gave reserved seats to
the depressed classes

72. As per Census 2011 data, which is the most populated metro city in India?

a) Delhi
b) Mumbai
c) Chennai
d) Kolkata
Correct Answer: B
Your Answer: Unanswered
Explanation:

IASbaba
Web: http://www.iasbaba.com/ Page 54
Email: iasbabailp2016@gmail.com
Exam Title : Test Series 3

Email :

Exam URL : http://test.iasbaba.com

Solution (b)

· (CENSUS-2011) Population wise Metropolitans: Mumbai > Delhi > Kolkata > Chennai > Bangalore.

73. For a place, to be called a town, according to Indian Census, which of the following conditions is not
required.

a) A minimum population of 5000


b) Minimum population density of 400 persons per sq.km
c) 75% of male population should be engaged in Non- Agriculture work.
d) 25% of area should be under the manufacturing sector.
Correct Answer: D
Your Answer: Unanswered
Explanation:
Solution (d)

· According to census, the towns should have the following conditions.

· minimum population 5000

· minimum population density 400 persons per sq.km

· 75% of males in non-agro work

74. Yellow revolution in India refers to the growth of

a) Smuggling of drugs
b) Increase in solar power
c) Oil seed production
d) Jaundice outbreak
Correct Answer: C
Your Answer: Unanswered
Explanation:

IASbaba
Web: http://www.iasbaba.com/ Page 55
Email: iasbabailp2016@gmail.com
Exam Title : Test Series 3

Email :

Exam URL : http://test.iasbaba.com

Solution (c)

· The oilseeds production scenario in India has witnessed a dramatic turn. The country achieved a status
of ‘self sufficient and net exporter’ during early nineties, rising from the ‘net importer’ state, with a mere
annual production of nearly 11 million tonnes from the annual oilseed crops, until the year 1986-87. In a
span of just a decade, an all time record oilseeds production of 25 million tonnes from annual oilseed
crops was attained during 1996 ,97 . This transformation has been termed as “The Yellow Revolution”
and could be primarily attributed to the institutional support, particularly the set up of the Technology
Mission on Oilseeds in 1986. Significant outcome of the Mission and other related developments
included the following; the improved oilseeds production technology, the expansion in cultivated area,
and the price support policy. As a result, the gains made possible were none short of a revolution and
were rightly named as the yellow revolution.

75. Consider the following statements:


1. National Solar Mission target has been increased fivefold to 1,00,000 MW till 2022.
2. Indian Government had put a clean energy cess which was doubled to Rs 100 per ton of emission.
Which of the above statements are incorrect?

a) 1 only
b) 2 only
c) Both
d) None of the above
Correct Answer: D
Your Answer: Unanswered
Explanation:
Solution (d)

· India introduced the clean energy cess on coal in 2010 which very few countries have in the world. This
has been doubled to Rs.100 per tonne in 2014. The total collection so far under the National Clean
Energy Fund (NCEF) has crossed Rs. 17,000 crores and till September, 2014, 46 clean energy projects
worth Rs.16 ,511.43 crores have been recommended for funding out of this fund.

· The Union Cabinet chaired by the Prime Minister, gave its approval for stepping up of India’s solar
power capacity target under the Jawaharlal Nehru National Solar Mission (JNNSM) by five times,
reaching 1,00,000 MW by 2022. The target will principally comprise of 40 GW Rooftop and 60 GW
through Large and Medium Scale Grid Connected Solar Power Projects. With this ambitious target, India
will become one of the largest Green Energy producers in the world, surpassing several developed
countries.

IASbaba
Web: http://www.iasbaba.com/ Page 56
Email: iasbabailp2016@gmail.com
Exam Title : Test Series 3

Email :

Exam URL : http://test.iasbaba.com

76. Consider the following regarding Diamond Quadrilateral:


1. The quadrilateral will connect the four metro cities – Delhi, Mumbai, Bangalore and Kolkata.
2. This is a project of Indian railways to establish a high speed rail network.
3. The corridor will be operated on broad guage tracks.
Select the correct option

a) 1 only
b) 2 and 3
c) 1 and 3
d) All of the above
Correct Answer: B
Your Answer: Unanswered
Explanation:
Solution (b)

· The Diamond Quadrilateral is a project of the Indian railways to establish high speed rail network in
India. This quadrilateral will connect the four metro cities in India,
i.e. Delhi, Mumbai, Chennai and Kolkata. This project is similar to Golden Quadrilateral which is a
roadway project which connects the four metros by Express Ways. The Golden Quadrilateral falls
under National Highways Development Project which has helped to build better road transport in India.
Similarly to improve country's rail infrastructure there is a need to implement High-speed trains / Bullet
trains.

· This corridor will be operated on broad-gauge tracks. The major terminals will be
at Delhi, Mumbai, Pune ,Thiruvananthapuram , Bangalore, Chennai, Hyderabad and Kolkata. The
corridor will pass through 14 states and territories in India. These states are Andhra
Pradesh, Bihar, Haryana, Jharkhand, Maharashtra, Karnataka, Kerala, Odisha , Rajasthan ,Tamil Nadu,
Telangana , Uttar Pradesh and West Bengal.

77. Select the incorrect option

a) Ozone in stratosphere acts as a protective layer while ozone in lower atmosphere acts as a pollutant
b) Ozone hole has been found only at lower latitudes.
c) Montreal protocol has been adopted to ban ozone depleting substances.
d) UV radiations can be harmful to living tissues.
Correct Answer: B
Your Answer: Unanswered
Explanation:

IASbaba
Web: http://www.iasbaba.com/ Page 57
Email: iasbabailp2016@gmail.com
Exam Title : Test Series 3

Email :

Exam URL : http://test.iasbaba.com

Solution (b)

· Ozone depletion refers to the phenomenon of reductions in the amount of ozone in the stratosphere.
The problem of ozone depletion is caused by high levels of chlorine and bromine compounds in the
stratosphere. The origins of these compounds are chlorofluorocarbons (CFC), used as cooling
substances in air conditioners and refrigerators, or as aerosol propellants, and bromofluoro carbons (
halons ), used in fire extinguishers. As a result of depletion of the ozone layer, more ultraviolet (UV)
radiation comes to Earth and causes damage to living organisms. UV radiation seems responsible for
skin cancer in humans; it also lowers production of phytoplankton and thus affects other aquatic
organisms. It can also influence the growth of terrestrial plants. A reduction of approximately 5 per cent
in the ozone layer was detected from1979 to 1990. Since the ozone layer prevents most harmful
wavelengths of ultraviolet light from passing through the Earth’s atmosphere, observed and projected
decreases in ozone have generated worldwide concern. This led to the adoption of the Montreal Protocol
banning the use of chlorofluorocarbon(CFC) compounds, as well as other ozone depleting chemicals
such as carbon tetrachloride, trichloroethane (also known as methyl chloroform), and bromine
,compounds known as halons .

· Ozone in lower atmosphere can cause chocking and photochemical smog. In large quantities it can
prove to be fatal.

· Because of the formation of polar vortex at poles and long winters, the ozone holes are only found at
poles and not on lower latitudes.

78. What is ‘dependency ratio’ for an economy?

a) The share of working age population as compared to the non-working population.


b) The share of non-working age population as compared to the working age population.
c) The ratio of working age population with that of the total population of the country.
d) None of the above.
Correct Answer: D
Your Answer: Unanswered
Explanation:
Solution (d)

· In economics, geography and demography the dependency ratio is an age-population ratio of those
typically not in the labor force (the dependent part) and those typically in the labor force
(the productive part). It is used to measure the pressure on productive population.

· In published international statistics, the dependent part usually includes those under the age of 15 and
over the age of 64.The productive part makes up the population in between, ages 15 – 64. It is normally
expressed as a percentage.

IASbaba
Web: http://www.iasbaba.com/ Page 58
Email: iasbabailp2016@gmail.com
Exam Title : Test Series 3

Email :

Exam URL : http://test.iasbaba.com

79. In Indian Five year plan periods, the Rolling Plan refers to

a) A new Five Year plan that was introduced by the Congress government in 1980
b) A New Plan introduced by the Janata government and subsequently rejected by the Congress
government
c) A two year interim plan introduced during the emergency period
d) None of the above
Correct Answer: B
Your Answer: Unanswered
Explanation:
Solution (b)

· Janta Govt. put forward a plan for 1978-1983. However, the government lasted for only 2 years.
Congress Govt. returned to power in 1980 and launched a different plan.

80. Consider the following statements about the impact of taxes on the economy of India?
1. It indirectly affects the level of investment in the economy.
2. Tweaking taxes can be counter-inflationary.
3. It has not effect on the foreign investments in the economy.
Which of these is/are true?

a) 1 and 2
b) 2 and 3
c) 1 and 3
d) All of the above
Correct Answer: A
Your Answer: Unanswered
Explanation:
Solution (a)

· Level of investment depends upon the money available in the economy.

· Fiscal measures can be taken to either absorb the money from the market or to increase the flow of
money in the market. Hence, taxes indirectly affect the level of investment in the country.

· Change in tax rates does have an effect on the foreign investment in the country. Very high taxes will
be counterproductive for a foreign firm. Hence can make them averse of the investment and vice versa.

IASbaba
Web: http://www.iasbaba.com/ Page 59
Email: iasbabailp2016@gmail.com
Exam Title : Test Series 3

Email :

Exam URL : http://test.iasbaba.com

81. The Micro Finance Institutions (MFIs) (Regulation and Development) Bill, 2012 intends to regulate and
develop the MFIs sector. Who can be the potential beneficiaries if it becomes a law?
1. Small and Marginal farmers
2. Self-Help Groups
3. MFIs
4. Local money lenders
Choose the correct answer using the codes below:

a) 1 and 2 only
b) 3 and 4 only
c) 1,2 and 3
d) All of the above
Correct Answer: C
Your Answer: Unanswered
Explanation:
Solution (c)

· Micro Finance institutions provide loans to Small and marginal farmers, SHGs and MFIs. Hence they
will be the beneficiaries.

· Since the lending rate of MFIs is very low, more and more people will turn to them rather than local
money lenders. Hence it will be adverse for local money lenders.

82. Which of the following ethnic minority community of Nepal is protesting for amendments in newly formed
constitution of Nepal adopted in September?

a) Khas-Chhetri
b) Tamang
c) Newars
d) Madhesi
Correct Answer: D
Your Answer: Unanswered
Explanation:

IASbaba
Web: http://www.iasbaba.com/ Page 60
Email: iasbabailp2016@gmail.com
Exam Title : Test Series 3

Email :

Exam URL : http://test.iasbaba.com

Solution (d)

· The Madhesi people inhabit the flat southern region of Nepal known as the Madhesh .

· According to the population census in 2011; Terai occupies 17% of the total area of Nepal and has 51%
of the Nepalese Population. Of the total population of Nepal, in Terai , 22% are Madhesis while 29% are
Non- Madhesis i.e. people from hills migrated to Terai . The region they inhabit is historically separate
from Nepal.

83. Which of the following is an acid-base indicator?

a) Vinegar
b) Lime water
c) Turmeric
d) Baking soda
Correct Answer: C
Your Answer: Unanswered
Explanation:
Solution (c)

Special types of substances are used to test whether a substance is acidic or basic. These substances
are known as indicators. The indicators change their colour when added to a solution containing an
acidic or a basic substance. Turmeric, litmus, china rose petals ( Gudhal ), etc., are some of the naturally
occurring indicators.

NCERT Science 7 th - Chapter 5

84. Consider the following statements with respect to recently developed Hybrid Vaccum toilets
1. The vacuum toilet evacuates its discharge into a biodigester tank that is fitted underneath the coach.
2. The biodegester tank contains aerobic bacteria that convert human fecal matter into water and small
amount of gases before discharging the same on the ground or track.
Select the correct answer using the code given below

a) 1 only
b) 2 only
c) Both 1 and 2
d) None of the above
Correct Answer: A
Your Answer: Unanswered
Explanation:

IASbaba
Web: http://www.iasbaba.com/ Page 61
Email: iasbabailp2016@gmail.com
Exam Title : Test Series 3

Email :

Exam URL : http://test.iasbaba.com

Solution (a)

Features of Hybrid Vacuum Toilet

· The prototype was built by modifying the standard flushing protocol of a vacuum toilet so as to create
water seal and additional post flush cycles.

· The vacuum toilet evacuates its discharge into a biodigester tank is fitted underneath the coach.

· The biodegester tank contains anaerobic bacteria that convert human faecal matter into water and
small amount of gases before discharging the same on the ground or track.

· Currently it is operated in Delhi Dibrugarh rajdhani express

· http://pib.nic.in/newsite/PrintRelease.aspx?relid=127005

85. Map of Africa

Identify the countries located in the Map

a) 1-Niger ,2-Somalia, 3-Burkino Faso ,4-Alegeria


b) 1-Somalia ,2-Niger, 3-Algeria, 4-Burkino Faso
c) 1-Algeria, 2-Niger,3-Somalia,4-Burkino Faso
d) 1-Burkino Faso,2-Somalia,3-Algeria,4-Niger
Correct Answer: C
Your Answer: Unanswered
Explanation:

IASbaba
Web: http://www.iasbaba.com/ Page 62
Email: iasbabailp2016@gmail.com
Exam Title : Test Series 3

Email :

Exam URL : http://test.iasbaba.com

Solution (c)

· 1-Algeria, 2-Niger,3-Somalia,4-Burkino Faso

· Burkino faso was in news in the month of September

· ATLAS

86. Which of the following keyword was not present in the original text of Preamble?

a) Sovereign
b) Justice
c) Socialist
d) Fraternity
Correct Answer: C
Your Answer: Unanswered
Explanation:
Solution (c)

· Controversy relating to the original text of Preamble was in news recently.

· 42nd Constitutional Amendment Act (1976) added three new words—socialist, secular and integrity.

· So the word socialist was not present in the original text of constitution

· Chap-4,Laxshmikanth Polity

87. Many of the sectoral schemes of the Government converge in ‘Smart Cities’. Consider the following
1. Atal Mission for Rejuvenation and Urban Transformation
2. Swachh Bharat Mission
3. Digital India
4. Indira Awas Yojana
5. INSPIRE Programme
Which of the above schemes of GoI converge in Smart Cities?

a) 1, 2 and 3
b) 1, 3 and 4
c) 2, 3 and 5
d) All
Correct Answer: A
Your Answer: Unanswered
Explanation:

IASbaba
Web: http://www.iasbaba.com/ Page 63
Email: iasbabailp2016@gmail.com
Exam Title : Test Series 3

Email :

Exam URL : http://test.iasbaba.com

Solution (a)

Under the Smart City Mission

§ The Central Government à Assistance to Urban Local Bodies

§ Technology backed urban centers

§ Promotion of a ‘bottom-up model’ of city development

Co nvergence of

§ AMRUT,

§ Swachh Bharat Mission (SBM),

§ National Heritage City Development and Augmentation Yojana (HRIDAY),

§ Digital India,

§ Skill Development,

§ Housing for All for holistic development of cities

http://smartcities.gov.in/writereaddata/Convergence.pdf and September 2015 Yojana

88. Which of the following are correct regarding MUDRA bank?


4. Its aim is to refinance the Micro Finance Institutions (MFIs) which are engaged in lending to MSMEs.
5. It will be responsible for accreditation and rating of MFIs
6. There are three financial stages for MUDRA- Shishu, Kishore and Tarun.
Select the correct option from the following

a) 1 only
b) 1 and 3
c) 2 and 3
d) All of the above
Correct Answer: D
Your Answer: Unanswered
Explanation:

IASbaba
Web: http://www.iasbaba.com/ Page 64
Email: iasbabailp2016@gmail.com
Exam Title : Test Series 3

Email :

Exam URL : http://test.iasbaba.com

Solution (d)

· MUDRA stands for Micro Units Development Refinance Agency (MUDRA). On 8th April 2015
government launched MUDRA bank.

· Aim – To refinance microfinance Institutions (MFIs) engaged in lending to MSME’s engaged in


manufacturing, trading and service activities.

· How will it achieve the objective –

· Laying down policy guidelines for MFIs

· Registration of MFIs

· Accreditation, rating of MFI

· Promoting technology

· Running a credit guarantee scheme for providing guarantees to loans

Other Facts –

· It will be the part of Small Industries Development Bank of India (SIDBI)

· Pradhan Mantri MUDRA Yojana will be the main support system for MUDRA Bank.

· Started with an initial corpus of Rs. 20,000 crore .

· There are 3 financing stages for MUDRA Bank known as

· Shishu : for start-ups. They are eligible for loan of Rs. 50,000.

· Kishore : entrepreneur will be eligible for a loan ranging from Rs 50,000 to Rs 5 lakh

· Tarun : maximum loan up to Rs. 10 lakh can be provided under this stage

89. Which of the following statements is correct with regard to the Preamble of the Indian Constitution?
1. The Preamble is a source of power to the legislature
2. Preamble is a part of the Constitution
3. Preamble cannot be amended
4. It is non-justiciable, or it is not enforceable in courts of law.
Select the correct answer using the code given below:
a) 2 and 4
b) 2 only
c) 1, 3 and 4

IASbaba
Web: http://www.iasbaba.com/ Page 65
Email: iasbabailp2016@gmail.com
Exam Title : Test Series 3

Email :

Exam URL : http://test.iasbaba.com

d) 1, 2 and 4
Correct Answer: A
Your Answer: Unanswered
Explanation:
Solution (a)

· In the Kesavanand Bharati case (1973), the Supreme Court held that Preamble is a part of the
Constitution and it can be amended, subject to the condition that no amendment is done to the ‘basic
features’. The Preamble has been amended only once so far, in 1976, by the 42nd Constitutional
Amendment Act, which has added three new words—Socialist, Secular and Integrity—to the Preamble.
The Preamble is neither a source of power to legislature nor a prohibition upon the powers of legislature.
It is non- justiciable , that is, its provisions are not enforceable in courts of law.

· Source: Laxmikanth Chapter 4

90. Cellulose-rich food substances are good source of roughage in human beings because

a) Human beings do not have cellulose-digesting enzymes.


b) Cellulose gets absorbed in the human blood and converts into fibres.
c) The cellulose-digesting bacteria convert cellulose into fibres.
d) Cellulose breaks down into smaller components which are egested as roughage.
Correct Answer: A
Your Answer: Unanswered
Explanation:
Solution (a)

NCERT Science 7 th - Chapter 2

91. With reference to Schedule 9 of the Indian constitution, consider the following
1. It consists of provisions relating to disqualification of members of parliament and state legislature on
ground of defection.
2. It was added by the 42nd Constitutional Amendment Act, 1976
3. The provisions under this schedule is subjected to Judicial Review
Choose the correct answer from the codes given below

a) 1 and 3 only
b) 1 and 2 only
c) 3 Only
d) None
Correct Answer: C

IASbaba
Web: http://www.iasbaba.com/ Page 66
Email: iasbabailp2016@gmail.com
Exam Title : Test Series 3

Email :

Exam URL : http://test.iasbaba.com

Your Answer: Unanswered


Explanation:
Solution (c)

· Schedule 9 contains list of acts and regulations of state legislatures which are beyond judicial review by
the courts.

· It was added through 1st constitutional amendment 1951.

· However, in 2007 the Supreme Court ruled that the laws included in this schedule after April 24, 1973
are now open to judicial review.

92. In which of the following ways a person can lose his citizenship of India immediately, during a war in
which India is engaged?
1. Renunciation
2. Termination
3. Deprivation
4. Naturalization
Select the correct answer using the code given below

a) 1, 2, 3 only
b) 3 only
c) 2, 3 only
d) All
Correct Answer: B
Your Answer: Unanswered
Explanation:

IASbaba
Web: http://www.iasbaba.com/ Page 67
Email: iasbabailp2016@gmail.com
Exam Title : Test Series 3

Email :

Exam URL : http://test.iasbaba.com

Solution (b)

· Any citizen of India of full age and capacity can make a declaration renouncing his Indian citizenship.
Upon the registration of that declaration, that person ceases to be a citizen of India. However, if such a
declaration is made during a war in which India is engaged, its registration shall be withheld by the
Central Government.

· Termination: When an Indian citizen voluntarily (consciously, knowingly and without duress, undue
influence or compulsion) acquires the citizenship of another country, his Indian citizenship automatically
terminates. This provision, however, does not apply during a war in which India is engaged.

· However it is compulsory to terminate a citizen ship if the citizen is unlawfully traded or communicated
with the enemy during the war

· Source: Laxmikanth chapter 6

93. With regard to Government of India Act 1935, consider the following statements
1. It introduced diarchy in provinces
2. The act divided the powers between the Centre under two lists namely federal list and provincial list.
3. It introduced bicameralism in the provinces
Choose the correct answer from the codes given below

a) 1 and 2
b) 3 Only
c) 2 and 3
d) None
Correct Answer: B
Your Answer: Unanswered
Explanation:

IASbaba
Web: http://www.iasbaba.com/ Page 68
Email: iasbabailp2016@gmail.com
Exam Title : Test Series 3

Email :

Exam URL : http://test.iasbaba.com

Solution (b)

· Important Provisions of GOI cat 1935

· It provided for establishment of All India Federation consisting of provinces and princely states.

· It abolished diarchy in provinces and tried to introduce in centre. However it was not introduced.

· The act divided the powers into federal list, provincial list and concurrent list.

· It provided for establishment of federal court and reserve bank of India.

· It abolished Indian council.

· It further extended principle of communal representation.

· It introduced bicameralism in six out of eleven provinces. Thus, the legislatures of Bengal, Bombay,
Madras, Bihar, Assam and the United Provinces were made bicameral consisting of a legislative council
(upper house) and a legislative assembly (lower house). However, many restrictions were placed on
them.

· It provided for the adoption of dyarchy at the Centre.

· It introduced for the first time bicameralism and GOI Act 1919

94. Which of the following is not a source of data on unemployment?

a) Reports of census of India


b) National Sample Survey Organisation
c) Directorate general for Employment and training
d) Central Statistical office
Correct Answer: D
Your Answer: Unanswered
Explanation:
Solution (d)

· There are three sources of data on unemployment: Reports of Census of India, National Sample
Survey Organisation’s Reports of Employment and Unemployment Situation and Directorate General of
Employment and Training

95. When water and nutrients are available, algae grow and multiply rapidly by?
a) Fragmentation
b) Eutrophication

IASbaba
Web: http://www.iasbaba.com/ Page 69
Email: iasbabailp2016@gmail.com
Exam Title : Test Series 3

Email :

Exam URL : http://test.iasbaba.com

c) Pollination
d) Budding

Correct Answer: A
Your Answer: Unanswered
Explanation:
Solution (a)

NCERT Science 7 th - Chapter 12

96. Select the incorrect statement

a) Arteries have thin elastic walls


b) The clot is formed because of the presence of another type of cells in the blood, called WBC
c) Veins are the vessels which carry oxygen-rich blood from all parts of the body back to the heart
d) All
Correct Answer: D
Your Answer: Unanswered
Explanation:
Solution (d)

Arteries carry oxygen-rich blood from the heart to all parts of the body. Since the blood flow is rapid and
at a high pressure, the arteries have thick elastic walls. The clot is formed because of the presence of
another type of cells in the blood, called platelets. Veins are the vessels which carry carbon dioxide-rich
blood from all parts of the body back to the heart. The veins have thin walls. There are valves present in
veins which allow blood to flow only towards the heart.

NCERT Science 7 th - Chapter 11

97. Read the following statements with reference to soil


1. Weathering is a very fast process of soil formation.
2. Percolation of water is faster in sandy soils.
3. Loamy soil contains only sand and clay.
4. Top soil contains the maximum amount of humus.
Choose the correct statements from the above.

a) 2 and 4
b) 1 and 3
c) 2 and 3
d) 1 and 2

IASbaba
Web: http://www.iasbaba.com/ Page 70
Email: iasbabailp2016@gmail.com
Exam Title : Test Series 3

Email :

Exam URL : http://test.iasbaba.com

Correct Answer: A
Your Answer: Unanswered
Explanation:
Solution (a)

NCERT Science 7 th - Chapter 9

98. Consider the following


1. Burning of coal
2. Burning of wood
3. Explosion of firework
4. Boiling of water
Which of the above are chemical changes?

a) 1 and 2
b) 2, 3 and 4
c) 3 and 4
d) 1, 2 and 3
Correct Answer: D
Your Answer: Unanswered
Explanation:
Solution (d)

Boiling of water is a physical change: NCERT Science 7 th - Chapter 6

99. When the soil is too basic, plants do not grow well in it. To improve its quality what must be added to the
soil?

a) Organic matter
b) Quick lime
c) Slaked lime
d) Calamine solution
Correct Answer: A
Your Answer: Unanswered
Explanation:
Solution (a)

NCERT Science 7 th - Chapter 5

100. Neutralization reaction is a

IASbaba
Web: http://www.iasbaba.com/ Page 71
Email: iasbabailp2016@gmail.com
Exam Title : Test Series 3

Email :

Exam URL : http://test.iasbaba.com

a) Physical and reversible change.


b) Physical change that cannot be reversed.
c) Chemical and reversible change.
d) Chemical change that cannot be reversed.
Correct Answer: D
Your Answer: Unanswered
Explanation:
Q.100) Solution (d)

When an acidic solution is mixed with a basic solution, both the solutions neutralize the effect of each
other. When an acid solution and a base solution are mixed in suitable amounts, both the acidic nature
of the acid and the basic nature of the base are destroyed. The resulting solution is neither acidic nor
basic.

NCERT Science 7 th - Chapter 5

IASbaba
Web: http://www.iasbaba.com/ Page 72
Email: iasbabailp2016@gmail.com

Vous aimerez peut-être aussi